| 701 | 
                AMC 12 1988 A Q1 | 
                \(\sqrt{8}+\sqrt{18}=\)
$$\textbf{(A)}\ \sqrt{2... | 
                1515.26 | 
            
            
            
                | 702 | 
                AMC 12 1988 A Q21 | 
                The complex number \(z\) satisfies \(z + |z| = 2 +... | 
                1515.26 | 
            
            
            
                | 703 | 
                AMC 12 1987 A Q6 | 
                In the \(\triangle ABC\) shown, \(D\) is some inte... | 
                1515.26 | 
            
            
            
                | 704 | 
                AMC 12 1981 A Q8 | 
                For all positive numbers \(x,y,z\) the product \((... | 
                1515.26 | 
            
            
            
                | 705 | 
                AMC 12 1996 A Q10 | 
                How many line segments have both their endpoints l... | 
                1515.26 | 
            
            
            
                | 706 | 
                AMC 12 1992 A Q21 | 
                For a finite sequence \(A = (a_1, a_2,\ldots,a_n)\... | 
                1515.26 | 
            
            
            
                | 707 | 
                AMC 12 1982 A Q7 | 
                If the operation \(x * y\) is defined by \(x * y =... | 
                1515.26 | 
            
            
            
                | 708 | 
                AMC 12 1982 A Q26 | 
                If the base \(8\) representation of a perfect squa... | 
                1515.26 | 
            
            
            
                | 709 | 
                AIME 2010 II Q13 | 
                The \( 52\) cards in a deck are numbered \( 1, 2, ... | 
                1515.26 | 
            
            
            
                | 710 | 
                AIME 2006 I Q15 | 
                Given that a sequence satisfies \(x_0=0\) and \(|x... | 
                1515.26 | 
            
            
            
                | 711 | 
                AMC 12 2016 B Q7 | 
                Josh writes the numbers \(1,2,3,\dots,99,100\). He... | 
                1515.24 | 
            
            
            
                | 712 | 
                AMC 10 2007 A Q22 | 
                A finite sequence of three-digit integers has the ... | 
                1515.23 | 
            
            
            
                | 713 | 
                AMC 12 1990 A Q27 | 
                Which of these triples could not be the lengths of... | 
                1515.23 | 
            
            
            
                | 714 | 
                AMC 10 2011 B Q11 | 
                There are \(52\) people in a room. What is the lar... | 
                1515.23 | 
            
            
            
                | 715 | 
                AMC 10 2013 A Q19 | 
                In base \(10\), the number \(2013\) ends in the di... | 
                1515.23 | 
            
            
            
                | 716 | 
                AMC 10 2007 A Q24 | 
                Circles centered at \( A\) and \( B\) each have ra... | 
                1515.23 | 
            
            
            
                | 717 | 
                AMC 12 2013 B Q17 | 
                Let \(a,b,\) and \(c\) be real numbers such that \... | 
                1515.20 | 
            
            
            
                | 718 | 
                AMC 8 2004 A Q5 | 
                The losing team of each game is eliminated from th... | 
                1515.18 | 
            
            
            
                | 719 | 
                AMC 8 1989 Q | 
                How many \(4 \times 4\) arrays whose entries are \... | 
                1515.17 | 
            
            
            
                | 720 | 
                AMC 10 2005 B Q21 | 
                Forty slips are placed into a hat, each bearing a ... | 
                1515.03 |